Docsity
Docsity

Prepare for your exams
Prepare for your exams

Study with the several resources on Docsity


Earn points to download
Earn points to download

Earn points by helping other students or get them with a premium plan


Guidelines and tips
Guidelines and tips

2023 Health Assessment Exam 2: Study Guide:Latest Update Guaranteed Success Best Rated A+, Exams of Health sciences

2023 Health Assessment Exam 2: Study Guide:Latest Update Guaranteed Success Best Rated A+

Typology: Exams

2022/2023

Available from 10/20/2023

dillon-cole
dillon-cole 🇺🇸

4.7

(3)

1.8K documents

1 / 146

Toggle sidebar

Related documents


Partial preview of the text

Download 2023 Health Assessment Exam 2: Study Guide:Latest Update Guaranteed Success Best Rated A+ and more Exams Health sciences in PDF only on Docsity! 2023 Health Assessment Exam 2: Study Guide:Latest Update Guaranteed Success Best Rated A+ Chapter 08: 1. When performing a physical assessment, the first technique the nurse will always use is: a. Palpation. b. Inspection. c. Percussion. d. Auscultation. B The skills requisite for the physical examination are inspection, palpation, percussion, and auscultation. The skills are performed one at a time and in this order (with the exception of the abdominal assessment, during which auscultation takes place before palpation and percussion). The assessment of each body system begins with inspection. A focused inspection takes time and yields a surprising amount of information. 2. The nurse is preparing to perform a physical assessment. Which statement is true about the physical assessment? The inspection phase: a. Usually yields little information. b. Takes time and reveals a surprising amount of information. c. May be somewhat uncomfortable for the expert practitioner. d. Requires a quick glance at the patient’s body systems before proceeding with palpation. B A focused inspection takes time and yields a surprising amount of information. Initially, the examiner may feel uncomfortable, staring at the person without also doing something. A focused assessment is significantly more than a “quick glance.” 3. The nurse is assessing a patient’s skin during an office visit. What part of the hand and technique should be used to best assess the patient’s skin temperature? a. Fingertips; they are more sensitive to small changes in temperature. b. Dorsal surface of the hand; the skin is thinner on this surface than on the palms. c. Ulnar portion of the hand; increased blood supply in this area enhances temperature sensitivity. d. Palmar surface of the hand; this surface is the most sensitive to temperature variations because of its increased nerve supply in this area. B The dorsa (backs) of the hands and fingers are best for determining temperature because the skin is thinner on the dorsal surfaces than on the palms. Fingertips are best for fine, tactile discrimination. The other responses are not useful for palpation. 4. Which of these techniques uses the sense of touch to assess texture, temperature, moisture, and swelling when the nurse is assessing a patient? a. Palpation b. Inspection c. Percussion d. Auscultation A Palpation uses the sense of touch to assess the patient for these factors. Inspection involves vision; percussion assesses through the use of palpable vibrations and audible sounds; and auscultation uses the sense of hearing. 5. The nurse is preparing to assess a patient’s abdomen by palpation. How should the nurse proceed? a. Palpation of reportedly “tender” areas are avoided because palpation in these areas may cause pain. b. Palpating a tender area is quickly performed to avoid any discomfort that the patient may experience. c. The assessment begins with deep palpation, while encouraging the patient to relax and to take deep breaths. d. The assessment begins with light palpation to detect surface characteristics and to accustom the patient to being touched. D Light palpation is initially performed to detect any surface characteristics and to accustom the person to being touched. Tender areas should be palpated last, not first. 6. The nurse would use bimanual palpation technique in which situation? a. Palpating the thorax of an infant a. Count the patient’s respirations. b. Bilaterally percuss the thorax, noting any differences in percussion tones. c. Call for a chest x-ray study, and wait for the results before beginning an assessment. d. Inspect the thorax for any new masses and bleeding associated with respirations. B Percussion is always available, portable, and offers instant feedback regarding changes in underlying tissue density, which may yield clues of the patient’s physical status. 13. The nurse is teaching a class on basic assessment skills. Which of these statements is true regarding the stethoscope and its use? a. Slope of the earpieces should point posteriorly (toward the occiput). b. Although the stethoscope does not magnify sound, it does block out extraneous room noise. c. Fit and quality of the stethoscope are not as important as its ability to magnify sound. d. Ideal tubing length should be 22 inches to dampen the distortion of sound. B The stethoscope does not magnify sound, but it does block out extraneous room sounds. The slope of the earpieces should point forward toward the examiner’s nose. Long tubing will distort sound. The fit and quality of the stethoscope are both important. 14. The nurse is preparing to use a stethoscope for auscultation. Which statement is true regarding the diaphragm of the stethoscope? The diaphragm: a. Is used to listen for high-pitched sounds. b. Is used to listen for low-pitched sounds. c. Should be lightly held against the person’s skin to block out low-pitched sounds. d. Should be lightly held against the person’s skin to listen for extra heart sounds and murmurs. A The diaphragm of the stethoscope is best for listening to high-pitched sounds such as breath, bowel, and normal heart sounds. It should be firmly held against the person’s skin, firmly enough to leave a ring. The bell of the stethoscope is best for soft, low-pitched sounds such as extra heart sounds or murmurs. 15. Before auscultating the abdomen for the presence of bowel sounds on a patient, the nurse should: a. Warm the endpiece of the stethoscope by placing it in warm water. b. Leave the gown on the patient to ensure that he or she does not get chilled during the examination. c. Ensure that the bell side of the stethoscope is turned to the “on” position. d. Check the temperature of the room, and offer blankets to the patient if he or she feels cold. D The examination room should be warm. If the patient shivers, then the involuntary muscle contractions can make it difficult to hear the underlying sounds. The end of the stethoscope should be warmed between the examiner’s hands, not with water. The nurse should never listen through a gown. The diaphragm of the stethoscope should be used to auscultate for bowel sounds. 16. The nurse will use which technique of assessment to determine the presence of crepitus, swelling, and pulsations? a. Palpation b. Inspection c. Percussion d. Auscultation A Palpation applies the sense of touch to assess texture, temperature, moisture, organ location and size, as well as any swelling, vibration or pulsation, rigidity or spasticity, crepitation, presence of lumps or masses, and the presence of tenderness or pain. 17. The nurse is preparing to use an otoscope for an examination. Which statement is true regarding the otoscope? The otoscope: a. Is often used to direct light onto the sinuses. b. Uses a short, broad speculum to help visualize the ear. c. Is used to examine the structures of the internal ear. d. Directs light into the ear canal and onto the tympanic membrane. D The otoscope directs light into the ear canal and onto the tympanic membrane that divides the external and middle ear. A short, broad speculum is used to visualize the nares. 18. An examiner is using an ophthalmoscope to examine a patient’s eyes. The patient has astigmatism and is nearsighted. The use of which of these techniques would indicate that the examination is being correctly performed? a. Using the large full circle of light when assessing pupils that are not dilated b. Rotating the lens selector dial to the black numbers to compensate for astigmatism c. Using the grid on the lens aperture dial to visualize the external structures of the eye d. Rotating the lens selector dial to bring the object into focus D The ophthalmoscope is used to examine the internal eye structures. It can compensate for nearsightedness or farsightedness, but it will not correct for astigmatism. The grid is used to assess size and location of lesions on the fundus. The large full spot of light is used to assess dilated pupils. Rotating the lens selector dial brings the object into focus. 19. The nurse is unable to palpate the right radial pulse on a patient. The best action would be to: a. Auscultate over the area with a fetoscope. b. Use a goniometer to measure the pulsations. c. Use a Doppler device to check for pulsations over the area. d. Check for the presence of pulsations with a stethoscope. C Doppler devices are used to augment pulse or blood pressure measurements. Goniometers measure joint range of motion. A fetoscope is used to auscultate fetal heart tones. Stethoscopes are used to auscultate breath, bowel, and heart sounds. 20. The nurse is preparing to perform a physical assessment. The correct action by the nurse is reflected by which statement? The nurse: c. “Your pulse is 80 beats per minute, which is within the normal range.” d. “I’m using my stethoscope to listen for any crackles, wheezes, or rubs.” C The sharing of some information builds rapport, as long as the patient is able to understand the terminology. 25. The nurse keeps in mind that the most important reason to share information and to offer brief teaching while performing the physical examination is to help the: a. Examiner feel more comfortable and to gain control of the situation. b. Examiner to build rapport and to increase the patient’s confidence in him or her. c. Patient understand his or her disease process and treatment modalities. d. Patient identify questions about his or her disease and the potential areas of patient education. B Sharing information builds rapport and increases the patient’s confidence in the examiner. It also gives the patient a little more control in a situation during which feeling completely helpless is often present. 26. The nurse is examining an infant and prepares to elicit the Moro reflex at which time during the examination? a. When the infant is sleeping b. At the end of the examination c. Before auscultation of the thorax d. Halfway through the examination B The Moro or startle reflex is elicited at the end of the examination because it may cause the infant to cry. 27. When preparing to perform a physical examination on an infant, the nurse should: a. Have the parent remove all clothing except the diaper on a boy. b. Instruct the parent to feed the infant immediately before the examination. c. Encourage the infant to suck on a pacifier during the abdominal examination. d. Ask the parent to leave the room briefly when assessing the infant’s vital signs. A The parent should always be present to increase the child’s feeling of security and to understand normal growth and development. The timing of the examination should be 1 to 2 hours after feeding when the baby is neither too drowsy nor too hungry. Infants do not object to being nude; clothing should be removed, but a diaper should be left on a boy. 28. A 6-month-old infant has been brought to the well-child clinic for a check-up. She is currently sleeping. What should the nurse do first when beginning the examination? a. Auscultate the lungs and heart while the infant is still sleeping. b. Examine the infant’s hips, because this procedure is uncomfortable. c. Begin with the assessment of the eye, and continue with the remainder of the examination in a head-to-toe approach. d. Wake the infant before beginning any portion of the examination to obtain the most accurate assessment of body systems. A When the infant is quiet or sleeping is an ideal time to assess the cardiac, respiratory, and abdominal systems. Assessment of the eye, ear, nose, and throat are invasive procedures that should be performed at the end of the examination. 29. A 2-year-old child has been brought to the clinic for a well-child checkup. The best way for the nurse to begin the assessment is to: a. Ask the parent to place the child on the examining table. b. Have the parent remove all of the child’s clothing before the examination. c. Allow the child to keep a security object such as a toy or blanket during the examination. d. Initially focus the interactions on the child, essentially ignoring the parent until the child’s trust has been obtained. C The best place to examine the toddler is on the parent’s lap. Toddlers understand symbols; therefore, a security object is helpful. Initially, the focus is more on the parent, which allows the child to adjust gradually and to become familiar with you. A 2-year-old child does not like to take off his or her clothes. Therefore, ask the parent to undress one body part at a time. 30. The nurse is examining a 2-year-old child and asks, “May I listen to your heart now?” Which critique of the nurse’s technique is most accurate? a. Asking questions enhances the child’s autonomy b. Asking the child for permission helps develop a sense of trust c. This question is an appropriate statement because children at this age like to have choices d. Children at this age like to say, “No.” The examiner should not offer a choice when no choice is available D Children at this age like to say, “No.” Choices should not be offered when no choice is really available. If the child says, “No” and the nurse does it anyway, then the nurse loses trust. Autonomy is enhanced by offering a limited option, “Shall I listen to your heart next or your tummy?” 31. With which of these patients would it be most appropriate for the nurse to use games during the assessment, such as having the patient “blow out” the light on the penlight? a. Infant b. Preschool child c. School-age child d. Adolescent B When assessing preschool children, using games or allowing them to play with the equipment to reduce their fears can be helpful. Such games are not appropriate for the other age groups. 32. The nurse is preparing to examine a 4-year-old child. Which action is appropriate for this age group? a. Explain the procedures in detail to alleviate the child’s anxiety. regardless of their risk or presumed infection status. Standard Precautions apply to blood and all other body fluids, secretions and excretions except sweat—regardless of whether they contain visible blood, nonintact skin, or mucous membranes. Hands should be washed with soap and water if visibly soiled with blood or body fluids. Alcohol-based hand rubs can be used if hands are not visibly soiled. 37. The nurse is preparing to assess a hospitalized patient who is experiencing significant shortness of breath. How should the nurse proceed with the assessment? a. The patient should lie down to obtain an accurate cardiac, respiratory, and abdominal assessment. b. A thorough history and physical assessment information should be obtained from the patient’s family member. c. A complete history and physical assessment should be immediately performed to obtain baseline information. d. Body areas appropriate to the problem should be examined and then the assessment completed after the problem has resolved. D Both altering the position of the patient during the examination and collecting a mini database by examining the body areas appropriate to the problem may be necessary in this situation. An assessment may be completed later after the distress is resolved. 38. When examining an infant, the nurse should examine which area first? a. Ear b. Nose c. Throat d. Abdomen D The least-distressing steps are performed first, saving the invasive steps of the examination of the eye, ear, nose, and throat until last. 39. While auscultating heart sounds, the nurse hears a murmur. Which of these instruments should be used to assess this murmur? a. Electrocardiogram b. Bell of the stethoscope c. Diaphragm of the stethoscope d. Palpation with the nurse’s palm of the hand B The bell of the stethoscope is best for soft, low-pitched sounds such as extra heart sounds or murmurs. The diaphragm of the stethoscope is best used for high-pitched sounds such as breath, bowel, and normal heart sounds. 40. During an examination of a patient’s abdomen, the nurse notes that the abdomen is rounded and firm to the touch. During percussion, the nurse notes a drumlike quality of the sounds across the quadrants. This type of sound indicates: a. Constipation. b. Air-filled areas. c. Presence of a tumor. d. Presence of dense organs. B A musical or drumlike sound (tympany) is heard when percussion occurs over an air- filled viscus, such as the stomach or intestines. 41. The nurse is preparing to examine a 6-year-old child. Which action is most appropriate? a. The thorax, abdomen, and genitalia are examined before the head. b. Talking about the equipment being used is avoided because doing so may increase the child’s anxiety. c. The nurse should keep in mind that a child at this age will have a sense of modesty. d. The child is asked to undress from the waist up. C A 6-year-old child has a sense of modesty. The child should undress him or herself, leaving underpants on and using a gown or drape. A school-age child is curious to know how equipment works, and the sequence should progress from the child’s head to the toes. 42. During auscultation of a patient’s heart sounds, the nurse hears an unfamiliar sound. The nurse should: a. Document the findings in the patient’s record. b. Wait 10 minutes, and auscultate the sound again. c. Ask the patient how he or she is feeling. d. Ask another nurse to double check the finding. D If an abnormal finding is not familiar, then the nurse may ask another examiner to double check the finding. The other responses do not help identify the unfamiliar sound. MULTIPLE RESPONSE 1. The nurse is preparing to palpate the thorax and abdomen of a patient. Which of these statements describes the correct technique for this procedure? Select all that apply. a. Warm the hands first before touching the patient. b. For deep palpation, use one long continuous palpation when assessing the liver. c. Start with light palpation to detect surface characteristics. d. Use the fingertips to examine skin texture, swelling, pulsation, and presence of lumps. e. Identify any tender areas, and palpate them last. f. Use the palms of the hands to assess temperature of the skin. A, C, D, E The hands should always be warmed before beginning palpation. Intermittent pressure rather than one long continuous palpation is used; any tender areas are identified and palpated last. Fingertips are used to examine skin texture, swelling, pulsation, and the presence of lumps. The dorsa (backs) of the hands are used to assess skin temperature because the skin on the dorsa is thinner than on the palms. c. Presence of kyphosis and flexion in the knees and hips d. Change in overall body proportion, including a longer trunk and shorter extremities C Changes that occur in the aging person include more prominent bony landmarks, decreased body weight (especially in men), a decrease in subcutaneous fat from the face and periphery, and additional fat deposited on the abdomen and hips. Postural changes of kyphosis and slight flexion in the knees and hips also occur. 7. The nurse should measure rectal temperatures in which of these patients? a. School-age child b. Older adult c. Comatose adult d. Patient receiving oxygen by nasal cannula C Rectal temperatures should be taken when the other routes are impractical, such as for comatose or confused persons, for those in shock, or for those who cannot close the mouth because of breathing or oxygen tubes, a wired mandible, or other facial dysfunctions. 8. The nurse is preparing to measure the length, weight, chest, and head circumference of a 6- month-old infant. Which measurement technique is correct? a. Measuring the infant’s length by using a tape measure b. Weighing the infant by placing him or her on an electronic standing scale c. Measuring the chest circumference at the nipple line with a tape measure d. Measuring the head circumference by wrapping the tape measure over the nose and cheekbones C To measure the chest circumference, the tape is encircled around the chest at the nipple line. The length should be measured on a horizontal measuring board. Weight should be measured on a platform-type balance scale. Head circumference is measured with the tape around the head, aligned at the eyebrows, and at the prominent frontal and occipital bones—the widest span is correct. 9. The nurse knows that one advantage of the tympanic membrane thermometer (TMT) is that: a. Rapid measurement is useful for uncooperative younger children. b. Using the TMT is the most accurate method for measuring body temperature in newborn infants. c. Measuring temperature using the TMT is inexpensive. d. Studies strongly support the use of the TMT in children under the age 6 years. A The TMT is useful for young children who may not cooperate for oral temperatures and fear rectal temperatures. However, the use a TMT with newborn infants and young children is conflicting. 10. When assessing an older adult, which vital sign changes occur with aging? a. Increase in pulse rate b. Widened pulse pressure c. Increase in body temperature d. Decrease in diastolic blood pressure B With aging, the nurse keeps in mind that the systolic blood pressure increases, leading to widened pulse pressure. With many older people, both the systolic and diastolic pressures increase. The pulse rate and temperature do not increase. 11. The nurse is examining a patient who is complaining of “feeling cold.” Which is a mechanism of heat loss in the body? a. Exercise b. Radiation c. Metabolism d. Food digestion B The body maintains a steady temperature through a thermostat or feedback mechanism, which is regulated in the hypothalamus of the brain. The hypothalamus regulates heat production from metabolism, exercise, food digestion, and external factors with heat loss through radiation, evaporation of sweat, convection, and conduction. 12. When measuring a patient’s body temperature, the nurse keeps in mind that body temperature is influenced by: a. Constipation. b. Patient’s emotional state. c. Diurnal cycle. d. Nocturnal cycle. C Normal temperature is influenced by the diurnal cycle, exercise, and age. The other responses do not influence body temperature. 13. When evaluating the temperature of older adults, the nurse should remember which aspect about an older adult’s body temperature? a. The body temperature of the older adult is lower than that of a younger adult. b. An older adult’s body temperature is approximately the same as that of a young child. c. Body temperature depends on the type of thermometer used. d. In the older adult, the body temperature varies widely because of less effective heat control mechanisms. A In older adults, the body temperature is usually lower than in other age groups, with a mean temperature of 36.2° C. 14. A 60-year-old male patient has been treated for pneumonia for the past 6 weeks. He is seen today in the clinic for an “unexplained” weight loss of 10 pounds over the last 6 weeks. The nurse knows that: a. Weight loss is probably the result of unhealthy eating habits. b. Chronic diseases such as hypertension cause weight loss. c. Unexplained weight loss often accompanies short-term illnesses. d. Weight loss is probably the result of a mental health dysfunction. C An unexplained weight loss may be a sign of a short-term illness or a chronic illness such as endocrine disease, malignancy, depression, anorexia nervosa, or bulimia. 15. When assessing a 75-year-old patient who has asthma, the nurse notes that he assumes a b. Pallor c. Capillary refill time d. Timing in the cardiac cycle A The pulse is assessed for rate, rhythm, and force. 21. When assessing the pulse of a 6-year-old boy, the nurse notices that his heart rate varies with his respiratory cycle, speeding up at the peak of inspiration and slowing to normal with expiration. The nurse’s next action would be to: a. Immediately notify the physician. b. Consider this finding normal in children and young adults. c. Check the child’s blood pressure, and note any variation with respiration. d. Document that this child has bradycardia, and continue with the assessment. B Sinus arrhythmia is commonly found in children and young adults. During the respiratory cycle, the heart rate varies, speeding up at the peak of inspiration and slowing to normal with expiration. 22. When assessing the force, or strength, of a pulse, the nurse recalls that the pulse: a. Is usually recorded on a 0- to 2-point scale. b. Demonstrates elasticity of the vessel wall. c. Is a reflection of the heart’s stroke volume. d. Reflects the blood volume in the arteries during diastole. C The heart pumps an amount of blood (the stroke volume) into the aorta. The force flares the arterial walls and generates a pressure wave, which is felt in the periphery as the pulse. 23. The nurse is assessing the vital signs of a 20-year-old male marathon runner and documents the following vital signs: temperature–36° C; pulse–48 beats per minute; respirations–14 breaths per minute; blood pressure–104/68 mm Hg. Which statement is true concerning these results? a. The patient is experiencing tachycardia. b. These are normal vital signs for a healthy, athletic adult. c. The patient’s pulse rate is not normal—his physician should be notified. d. On the basis of these readings, the patient should return to the clinic in 1 week. B In the adult, a heart rate less than 50 beats per minute is called bradycardia, which normally occurs in the well-trained athlete whose heart muscle develops along with the skeletal muscles. 24. The nurse is assessing the vital signs of a 3-year-old patient who appears to have an irregular respiratory pattern. How should the nurse assess this child’s respirations? a. Respirations should be counted for 1 full minute, noticing rate and rhythm. b. Child’s pulse and respirations should be simultaneously checked for 30 seconds. c. Child’s respirations should be checked for a minimum of 5 minutes to identify any variations in his or her respiratory pattern. d. Patient’s respirations should be counted for 15 seconds and then multiplied by 4 to obtain the number of respirations per minute. A Respirations are counted for 1 full minute if an abnormality is suspected. The other responses are not correct actions. 25. A patient’s blood pressure is 118/82 mm Hg. He asks the nurse, “What do the numbers mean?” The nurse’s best reply is: a. “The numbers are within the normal range and are nothing to worry about.” b. “The bottom number is the diastolic pressure and reflects the stroke volume of the heart.” c. “The top number is the systolic blood pressure and reflects the pressure of the blood against the arteries when the heart contracts.” d. “The concept of blood pressure is difficult to understand. The primary thing to be concerned about is the top number, or the systolic blood pressure.” C The systolic pressure is the maximum pressure felt on the artery during left ventricular contraction, or systole. The diastolic pressure is the elastic recoil, or resting, pressure that the blood constantly exerts in between each contraction. The nurse should answer the patient’s question and use terms he can understand. 26. While measuring a patient’s blood pressure, the nurse recalls that certain factors, such as , help determine blood pressure. a. Pulse rate b. Pulse pressure c. Vascular output d. Peripheral vascular resistance ANS: The level of blood pressure is determined by five factors: cardiac output, peripheral vascular resistance, volume of circulating blood, viscosity, and elasticity of the vessel walls. 27. A nurse is helping at a health fair at a local mall. When taking blood pressures on a variety of people, the nurse keeps in mind that: a. After menopause, blood pressure readings in women are usually lower than those taken in men. b. The blood pressure of a Black adult is usually higher than that of a White adult of the same age. c. Blood pressure measurements in people who are overweight should be the same as those of people who are at a normal weight. d. A teenager’s blood pressure reading will be lower than that of an adult. B In the United States, a Black adult’s blood pressure is usually higher than that of a White adult of the same age. The incidence of hypertension is twice as high in Blacks as it is in Whites. After menopause, blood pressure in women is higher than in men; blood pressure measurements in people who are obese are usually higher than in those who are not overweight. Normally, a gradual rise occurs through childhood and into the adult years. a. 200/92 b. 200/100 c. 100/200/92 d. 200/100/92 A In adults, the last audible sound best indicates the diastolic pressure. When a variance is greater than 10 to 12 mm Hg between phases IV and V, both phases should be recorded along with the systolic reading (e.g., 142/98/80). 34. A patient is seen in the clinic for complaints of “fainting episodes that started last week.” How should the nurse proceed with the examination? a. Blood pressure readings are taken in both the arms and the thighs. b. The patient is assisted to a lying position, and his blood pressure is taken. c. His blood pressure is recorded in the lying, sitting, and standing positions. d. His blood pressure is recorded in the lying and sitting positions; these numbers are then averaged to obtain a mean blood pressure. C If the person is known to have hypertension, is taking antihypertensive medications, or reports a history of fainting or syncope, then the blood pressure reading should be taken in three positions: lying, sitting, and standing. 35. A 70-year-old man has a blood pressure of 150/90 mm Hg in a lying position, 130/80 mm Hg in a sitting position, and 100/60 mm Hg in a standing position. How should the nurse evaluate these findings? a. These readings are a normal response and attributable to changes in the patient’s position. b. The change in blood pressure readings is called orthostatic hypotension. c. The blood pressure reading in the lying position is within normal limits. d. The change in blood pressure readings is considered within normal limits for the patient’s age. B Orthostatic hypotension is a drop in systolic pressure of more than 20 mm Hg, which occurs with a quick change to a standing position. Aging people have the greatest risk of this problem. 36. The nurse is helping another nurse to take a blood pressure reading on a patient’s thigh. Which action is correct regarding thigh pressure? a. Either the popliteal or femoral vessels should be auscultated to obtain a thigh pressure. b. The best position to measure thigh pressure is the supine position with the knee slightly bent. c. If the blood pressure in the arm is high in an adolescent, then it should be compared with the thigh pressure. d. The thigh pressure is lower than the pressure in the arm, which is attributable to the distance away from the heart and the size of the popliteal vessels. C When blood pressure measured at the arm is excessively high, particularly in adolescents and young adults, it is compared with thigh pressure to check for coarctation of the aorta. The popliteal artery is auscultated for the reading. Generally, thigh pressure is higher than that of the arm; however, if coarctation of the artery is present, then arm pressures are higher than thigh pressures. 37. The nurse is preparing to measure the vital signs of a 6-month-old infant. Which action by the nurse is correct? a. Respirations are measured; then pulse and temperature. b. Vital signs should be measured more frequently than in an adult. c. Procedures are explained to the parent, and the infant is encouraged to handle the equipment. d. The nurse should first perform the physical examination to allow the infant to become more familiar with her and then measure the infant’s vital signs. A With an infant, the order of vital sign measurements is reversed to respiration, pulse, and temperature. Taking the temperature first, especially if it is rectal, may cause the infant to cry, which will increase the respiratory and pulse rate, thus masking the normal resting values. The vital signs are measured with the same purpose and frequency as would be measured in an adult. 38. A 4-month-old child is at the clinic for a well-baby check-up and immunizations. Which of these actions is most appropriate when the nurse is assessing an infant’s vital signs? a. The infant’s radial pulse should be palpated, and the nurse should notice any fluctuations resulting from activity or exercise. b. The nurse should auscultate an apical rate for 1 minute and then assess for any normal irregularities, such as sinus arrhythmia. c. The infant’s blood pressure should be assessed by using a stethoscope with a large diaphragm piece to hear the soft muffled Korotkoff sounds. d. The infant’s chest should be observed and the respiratory rate counted for 1 minute; the respiratory pattern may vary significantly. B The nurse palpates or auscultates an apical rate with infants and toddlers. The pulse should be counted for 1 full minute to account for normal irregularities, such as sinus arrhythmia. Children younger than 3 years of age have such small arm vessels; consequently, hearing Korotkoff sounds with a stethoscope is difficult. The nurse should use either an electronic blood pressure device that uses oscillometry or a Doppler ultrasound device to amplify the sounds. 39. The nurse is conducting a health fair for older adults. Which statement is true regarding vital sign measurements in aging adults? a. The pulse is more difficult to palpate because of the stiffness of the blood vessels. b. An increased respiratory rate and a shallower inspiratory phase are expected findings. c. A decreased pulse pressure occurs from changes in the systolic and diastolic blood pressures. d. Changes in the body’s temperature C MAP is the pressure that forces blood into the tissues, averaged over the cardiac cycle. Stroke volume is reflected by the blood pressure. MAP is not an arithmetic average of systolic and diastolic pressures because diastole lasts longer; rather, it is a value closer to diastolic pressure plus one third of the pulse pressure. 45. A 75-year-old man with a history of hypertension was recently changed to a new antihypertensive drug. He reports feeling dizzy at times. How should the nurse evaluate his blood pressure? a. Blood pressure and pulse should be recorded in the supine, sitting, and standing positions. b. The patient should be directed to walk around the room and his blood pressure assessed after this activity. c. Blood pressure and pulse are assessed at the beginning and at the end of the examination. d. Blood pressure is taken on the right arm and then 5 minutes later on the left arm. A Orthostatic vital signs should be taken when the person is hypertensive or is taking antihypertensive medications, when the person reports fainting or syncope, or when volume depletion is suspected. The blood pressure and pulse readings are recorded in the supine, sitting, and standing positions. 46. Which of these specific measurements is the best index of a child’s general health? a. Vital signs b. Height and weight c. Head circumference d. Chest circumference B Physical growth, measured by height and weight, is the best index of a child’s general health. 47. The nurse is assessing an 8-year-old child whose growth rate measures below the third percentile for a child his age. He appears significantly younger than his stated age and is chubby with infantile facial features. Which condition does this child have? a. Hypopituitary dwarfism b. Achondroplastic dwarfism c. Marfan syndrome d. Acromegaly A Hypopituitary dwarfism is caused by a deficiency in growth hormone in childhood and results in a retardation of growth below the third percentile, delayed puberty, and other problems. The child’s appearance fits this description. Achondroplastic dwarfism is a genetic disorder resulting in characteristic deformities; Marfan syndrome is an inherited connective tissue disorder characterized by a tall, thin stature and other features. Acromegaly is the result of excessive secretion of growth hormone in adulthood. (For more information, see Table 9-5, Abnormalities in Body Height and Proportion.) 48. The nurse is counting an infant’s respirations. Which technique is correct? a. Watching the chest rise and fall b. Watching the abdomen for movement c. Placing a hand across the infant’s chest d. Using a stethoscope to listen to the breath sounds B Watching the abdomen for movement is the correct technique because the infant’s respirations are normally more diaphragmatic than thoracic. The other responses do not reflect correct techniques. 49. When checking for proper blood pressure cuff size, which guideline is correct? a. The standard cuff size is appropriate for all sizes. b. The length of the rubber bladder should equal 80% of the arm circumference. c. The width of the rubber bladder should equal 80% of the arm circumference. d. The width of the rubber bladder should equal 40% of the arm circumference. D The width of the rubber bladder should equal 40% of the circumference of the person's arm. The length of the bladder should equal 80% of this circumference. 50. During an examination, the nurse notices that a female patient has a round “moon” face, central trunk obesity, and a cervical hump. Her skin is fragile with bruises. The nurse determines that the patient has which condition? a. Marfan syndrome b. Gigantism c. Cushing syndrome d. Acromegaly C Cushing syndrome is characterized by weight gain and edema with central trunk and cervical obesity (buffalo hump) and round plethoric face (moon face). Excessive catabolism causes muscle wasting; weakness; thin arms and legs; reduced height; and thin, fragile skin with purple abdominal striae, bruising, and acne. (See Table 9-5, Abnormalities in Body Height and Proportion, for the definitions of the other conditions.) MULTIPLE RESPONSE 1. While measuring a patient’s blood pressure, the nurse uses the proper technique to obtain an accurate reading. Which of these situations will result in a falsely high blood pressure reading? Select all that apply. a. The person supports his or her own arm during the blood pressure reading. b. The blood pressure cuff is too narrow for the extremity. c. The arm is held above level of the heart. d. The cuff is loosely wrapped around the arm. e. The person is sitting with his or her legs crossed. f. The nurse does not inflate the cuff high enough. A, B, D, E Several factors can result in blood pressure readings that are too high or too low. Having the patient’s arm held above the level of the heart is one part of the correct technique. (Refer to Table 9-5, Common Errors in Blood Pressure Measurement.) SHORT ANSWER 1. What is the pulse pressure for a patient whose blood pressure is 158/96 mm Hg and whose pulse rate is 72 beats per minute? 62 The pulse pressure is the difference between the systolic and diastolic and reflects the stroke volume. The pulse rate is not necessary for pulse pressure calculations. tomographic scan. d. Subjective report. D The subjective report is the most reliable indicator of pain. Physical examination findings can lend support, but the clinician cannot exclusively base the diagnosis of pain on physical assessment findings. 7. A patient has had arthritic pain in her hips for several years since a hip fracture. She is able to move around in her room and has not offered any complaints so far this morning. However, when asked, she states that her pain is “bad this morning” and rates it at an 8 on a 1-to-10 scale. What does the nurse suspect? The patient: a. Is addicted to her pain medications and cannot obtain pain relief. b. Does not want to trouble the nursing staff with her complaints. c. Is not in pain but rates it high to receive pain medication. d. Has experienced chronic pain for years and has adapted to it. D Persons with chronic pain typically try to give little indication that they are in pain and, over time, adapt to the pain. As a result, they are at risk for underdetection. 8. The nurse is reviewing the principles of pain. Which type of pain is due to an abnormal processing of the pain impulse through the peripheral or central nervous system? a. Visceral b. Referred c. Cutaneous d. Neuropathic D Neuropathic pain implies an abnormal processing of the pain message. The other types of pain are named according to their sources. 9. When assessing the quality of a patient’s pain, the nurse should ask which question? a. “When did the pain start?” b. “Is the pain a stabbing pain?” c. “Is it a sharp pain or dull pain?” d. “What does your pain feel like?” D To assess the quality of a person’s pain, the patient is asked to describe the pain in his or her own words. 10. When assessing a patient’s pain, the nurse knows that an example of visceral pain would be: a. Hip fracture. b. Cholecystitis. c. Second-degree burns. d. Pain after a leg amputation. B Visceral pain originates from the larger interior organs, such as the gallbladder, liver, or kidneys. 11. The nurse is reviewing the principles of nociception. During which phase of nociception does the conscious awareness of a painful sensation occur? a. Perception b. Modulation c. Transduction d. Transmission A Perception is the third phase of nociception and indicates the conscious awareness of a painful sensation. During this phase, the sensation is recognized by higher cortical structures and identified as pain. 12. When assessing the intensity of a patient’s pain, which question by the nurse is appropriate? a. “What makes your pain better or worse?” b. “How much pain do you have now?” c. “How does pain limit your activities?” d. “What does your pain feel like?” B Asking the patient “how much pain do you have?” is an assessment of the intensity of a patient’s pain; various intensity scales can be used. Asking what makes one’s pain better or worse assesses alleviating or aggravating factors. Asking whether pain limits one’s activities assesses the degree of impairment and quality of life. Asking “what does your pain feel like” assesses the quality of pain. 13. A patient is complaining of severe knee pain after twisting it during a basketball game and is requesting pain medication. Which action by the nurse is appropriate? a. Completing the physical examination first and then giving the pain medication b. Telling the patient that the pain medication must wait until after the x-ray images are completed c. Evaluating the full range of motion of the knee and then medicating for pain d. Administering pain medication and then proceeding with the assessment D According to the American Pain Society (1992), “In cases in which the cause of acute pain is uncertain, establishing a diagnosis is a priority, but symptomatic treatment of pain should be given while the investigation is proceeding. With occasional exceptions, (e.g., the initial examination of the patient with an acute condition of the abdomen), it is rarely justified to defer analgesia until a diagnosis is made. In fact, a comfortable patient is better able to cooperate with diagnostic procedures.” 14. The nurse knows that which statement is true regarding the pain experienced by infants? a. Pain in infants can only be assessed by physiologic changes, such as an increased heart rate. b. The FPS-R can be used to assess pain in infants. c. A procedure that induces pain in adults will also induce pain in the infant. d. Infants feel pain less than do adults. C If a procedure or disease process causes pain in an adult, then it will also cause pain in an infant. Physiologic changes cannot be exclusively used to confirm or deny pain because other factors, such as medications, fluid status, or stress may cause physiologic changes. The FPS-R can be used starting at age 4 years. 15. A patient has been admitted to the hospital with vertebral fractures related to osteoporosis. She is in extreme pain. This type of pain would be classified as: a. Referred. b. Cutaneous. c. Visceral. decreased. d. The baby should be placed on skim milk to decrease the risk of coronary artery disease when he or she grows older. A Because of rapid growth, especially of the brain, both infants and children younger than 2 years of age should not drink skim or low-fat milk or be placed on low-fat diets. Fats (calories and essential fatty acids) are required for proper growth and central nervous system development. 4. A pregnant woman is interested in breastfeeding her baby and asks several questions about the topic. Which information is appropriate for the nurse to share with her? a. Breastfeeding is best when also supplemented with bottle feedings. b. Babies who are breastfed often require supplemental vitamins. c. Breastfeeding is recommended for infants for the first 2 years of life. d. Breast milk provides the nutrients necessary for growth, as well as natural immunity. D Breastfeeding is recommended for full-term infants for the first year of life because breast milk is ideally formulated to promote normal infant growth and development, as well as natural immunity. The other statements are not correct. 5. A mother and her 13-year-old daughter express their concern related to the daughter’s recent weight gain and her increase in appetite. Which of these statements represents information the nurse should discuss with them? a. Dieting and exercising are necessary at this age. b. Snacks should be high in protein, iron, and calcium. c. Teenagers who have a weight problem should not be allowed to snack. d. A low-calorie diet is important to prevent the accumulation of fat. B After a period of slow growth in late childhood, adolescence is characterized by rapid physical growth and endocrine and hormonal changes. Caloric and protein requirements increase to meet this demand. Because of bone growth and increasing muscle mass (and, in girls, the onset of menarche), calcium and iron requirements also increase. 6. The nurse is assessing a 30-year-old unemployed immigrant from an underdeveloped country who has been in the United States for 1 month. Which of these problems related to his nutritional status might the nurse expect to find? a. Obesity b. Hypotension c. Osteomalacia (softening of the bones) d. Coronary artery disease C General undernutrition, hypertension, diarrhea, lactose intolerance, osteomalacia, scurvy, and dental caries are among the more common nutrition-related problems of new immigrants from developing countries. 7. For the first time, the nurse is seeing a patient who has no history of nutrition-related problems. The initial nutritional screening should include which activity? a. Calorie count of nutrients b. Anthropometric measures c. Complete physical examination d. Measurement of weight and weight history D Parameters used for nutrition screening typically include weight and weight history, conditions associated with increased nutritional risk, diet information, and routine laboratory data. The other responses reflect a more in-depth assessment rather than a screening. 8. A patient is asked to indicate on a form how many times he eats a specific food. This method describes which of these tools for obtaining dietary information? a. Food diary b. Calorie count c. 24-hour recall d. Food-frequency questionnaire D With this tool, information is collected on how many times per day, week, or month the individual eats particular foods, which provides an estimate of usual intake. 9. The nurse is providing care for a 68-year-old woman who is complaining of constipation. What concern exists regarding her nutritional status? a. Absorption of nutrients may be impaired. b. Constipation may represent a food allergy. c. The patient may need emergency surgery to correct the problem. d. Gastrointestinal problems will increase her caloric demand. A Gastrointestinal symptoms such as vomiting, diarrhea, or constipation may interfere with nutrient intake or absorption. The other responses are not correct. 10. During a nutritional assessment, why is it important for the nurse to ask a patient what medications he or she is taking? a. Certain drugs can affect the metabolism of nutrients. b. The nurse needs to assess the patient for allergic reactions. c. Medications need to be documented in the record for the physician’s review. d. Medications can affect one’s memory and ability to identify food eaten in the last 24 hours. A Analgesics, antacids, anticonvulsants, antibiotics, diuretics, laxatives, antineoplastic drugs, steroids, and oral contraceptives are drugs that can interact with nutrients, impairing their digestion, absorption, metabolism, or use. The other responses are not correct. 11. A patient tells the nurse that his food simply does not have any taste anymore. The nurse’s best response would be: a. “That must be really frustrating.” b. “When did you first notice this change?” c. “My food doesn’t always have a lot of taste either.” d. “Sometimes that happens, but your taste will come back.” B With changes in appetite, taste, smell, or chewing or swallowing, the examiner should ask about the type of change and when the change occurred. These problems interfere with adequate nutrient intake. The other responses are not correct. or her back to the examiner and arms folded across the chest. The skin on the forearm is pinched. C While holding the skinfold, the lever of the calipers is released. The nurse waits 3 seconds and then takes a reading. This procedure should be repeated three times, and an average of the three skinfold measurements is then recorded. 18. In teaching a patient how to determine total body fat at home, the nurse includes instructions to obtain measurements of: a. Height and weight. b. Frame size and weight. c. Waist and hip circumferences. d. Mid-upper arm circumference and arm span. A Body mass index, calculated by using height and weight measurements, is a practical marker of optimal weight for height and an indicator of obesity. The other options are not correct. 19. The nurse is evaluating patients for obesity-related diseases by calculating the waist-to- hip ratios. Which one of these patients would be at increased risk? a. 29-year-old woman whose waist measures 33 inches and hips measure 36 inches b. 32-year-old man whose waist measures 34 inches and hips measure 36 inches c. 38-year-old man whose waist measures 35 inches and hips measure 38 inches d. 46-year-old woman whose waist measures 30 inches and hips measure 38 inches A The waist-to-hip ratio assesses body fat distribution as an indicator of health risk. A waist-to-hip ratio of 1.0 or greater in men or 0.8 or greater in women is indicative of android (upper body obesity) and increasing risk for obesity-related disease and early death. The 29-year-old woman has a waist-to-hip ratio of 0.92, which is greater than 0.8. The 32-year-old man has a waist-to-hip ratio of 0.94; the 38-year-old man has a waist-to- hip ratio of 0.92; the 46-year-old woman has a waist-to-hip ratio of 0.78. 20. A 50-year-old woman with elevated total cholesterol and triglyceride levels is visiting the clinic to find out about her laboratory results. What would be important for the nurse to include in patient teaching in relation to these tests? a. The risks of undernutrition should be included. b. Offer methods to reduce the stress in her life. c. Provide information regarding a diet low in saturated fat. d. This condition is hereditary; she can do nothing to change the levels. C The patient with elevated cholesterol and triglyceride levels should be taught about eating a healthy diet that limits the intake of foods high in saturated fats or trans fats. Reducing dietary fats is part of the treatment for this condition. The other responses are not pertinent to her condition. 21. In performing an assessment on a 49-year-old woman who has imbalanced nutrition as a result of dysphagia, which data would the nurse expect to find? a. Increase in hair growth b. Inadequate nutrient food intake c. Weight 10% to 20% over ideal d. Sore, inflamed buccal cavity B Dysphagia, or impaired swallowing, interferes with adequate nutrient intake. 22. A 21-year-old woman has been on a low-protein liquid diet for the past 2 months. She has had adequate intake of calories and appears well nourished. After further assessment, what would the nurse expect to find? a. Poor skin turgor b. Decreased serum albumin c. Increased lymphocyte count d. Triceps skinfold less than standard B Kwashiorkor (protein malnutrition) is due to diets that may be high in calories but contain little or no protein (e.g., low-protein liquid diets, fad diets, and long-term use of dextrose- containing intravenous fluids). The serum albumin would be less than 3.5 g/dL. 23. The nurse is performing a nutritional assessment on an 80-year-old patient. The nurse knows that physiologic changes can directly affect the nutritional status of the older adult and include: a. Slowed gastrointestinal motility. b. Hyperstimulation of the salivary glands. c. Increased sensitivity to spicy and aromatic foods. d. Decreased gastrointestinal absorption causing esophageal reflux. A Normal physiologic changes in aging adults that affect nutritional status include slowed gastrointestinal motility, decreased gastrointestinal absorption, diminished olfactory and taste sensitivity, decreased saliva production, decreased visual acuity, and poor dentition. 24. Which of these interventions is most appropriate when the nurse is planning nutritional interventions for a healthy, active 74-year-old woman? a. Decreasing the amount of carbohydrates to prevent lean muscle catabolism b. Increasing the amount of soy and tofu in her diet to promote bone growth and reverse osteoporosis c. Decreasing the number of calories she is eating because of the decrease in energy requirements from the loss of lean body mass d. Increasing the number of calories she is eating because of the increased energy needs of the older adult C Important nutritional features of the older years are a decrease in energy requirements as a result of loss of lean body mass, the most metabolically active tissue, and an increase in fat mass. 25. A 16-year-old girl is being seen at the clinic for gastrointestinal complaints and weight loss. The nurse determines that many of her complaints may be related to erratic eating patterns, eating predominantly fast foods, and high caffeine intake. In this situation, which is most appropriate when collecting current dietary intake information? a. Scheduling a time for direct observation of the adolescent during meals b. Asking the patient for a 24-hour diet recall, and assuming it to be reflective of a typical day for her c. Having the patient complete a food diary for 3 days, including 2 weekdays and 1 weekend day current weight. B By dividing her current weight by her usual weight and then multiplying by 100, a percentage of 78.4% is obtained, which means that her current weight is 78.4% of her ideal body weight. A current weight of 80% to 90% of ideal weight suggests mild malnutrition; a current weight of 70% to 80% of ideal weight suggests moderate malnutrition; a current weight of less than 70% of ideal weight suggests severe malnutrition. MULTIPLE RESPONSE 1. The nurse is assessing a patient who is obese for signs of metabolic syndrome. This condition is diagnosed when three or more certain risk factors are present. Which of these assessment findings are risk factors for metabolic syndrome? Select all that apply. a. Fasting plasma glucose level less than 100 mg/dL b. Fasting plasma glucose level greater than or equal to 110 mg/dL c. Blood pressure reading of 140/90 mm Hg d. Blood pressure reading of 110/80 mm Hg e. Triglyceride level of 120 mg/dL B, C Metabolic syndrome is diagnosed when three or more of the following risk factors are present: (1) fasting plasma glucose level greater than or equal to 100 mg/dL; (2) blood pressure greater than or equal to 130/85 mm Hg; (3) waist circumference greater than or equal to 40 inches for men and 35 inches for women; (4) high-density lipoprotein cholesterol less than 40 in men and less than 50 in women; and (5) triglyceride levels greater than or equal to 150 mg/dL (ATP III, 2001). SHORT ANSWER 1. A patient has been unable to eat solid food for 2 weeks and is in the clinic today complaining of weakness, tiredness, and hair loss. The patient states that her usual weight is 175 pounds, but today she weighs 161 pounds. What is her recent weight change percentage? To calculate recent weight change percentage, use this formula: Usual weight – current weight  100 usual weight 8% 175 – 161 = 14 pounds 14 ÷ 175 = 0.08 0.08  100 = 8% Chapter 12: 1. The nurse educator is preparing an education module for the nursing staff on the epidermal layer of skin. Which of these statements would be included in the module? The epidermis is: a. Highly vascular. b. Thick and tough. c. Thin and nonstratified. d. Replaced every 4 weeks. D The epidermis is thin yet tough, replaced every 4 weeks, avascular, and stratified into several zones. 2. The nurse educator is preparing an education module for the nursing staff on the dermis layer of skin. Which of these statements would be included in the module? The dermis: a. Contains mostly fat cells. b. Consists mostly of keratin. c. Is replaced every 4 weeks. d. Contains sensory receptors. D The dermis consists mostly of collagen, has resilient elastic tissue that allows the skin to stretch, and contains nerves, sensory receptors, blood vessels, and lymphatic vessels. It is not replaced every 4 weeks. 3. The nurse is examining a patient who tells the nurse, “I sure sweat a lot, especially on my face and feet but it doesn’t have an odor.” The nurse knows that this condition could be related to: a. Eccrine glands. b. Apocrine glands. c. Disorder of the stratum corneum. d. Disorder of the stratum germinativum. A The eccrine glands are coiled tubules that directly open onto the skin surface and produce a dilute saline solution called sweat. Apocrine glands are primarily located in the axillae, anogenital area, nipples, and naval area and mix with bacterial flora to produce the characteristic musky body odor. The patient’s statement is not related to disorders of the stratum corneum or the stratum germinativum. 4. A newborn infant is in the clinic for a well-baby checkup. The nurse observes the infant for the possibility of fluid loss because of which of these factors? a. Subcutaneous fat deposits are high in the newborn. b. Sebaceous glands are overproductive in the newborn. c. The newborn’s skin is more permeable than that of the adult. d. The amount of vernix caseosa dramatically rises in the newborn. C The newborn’s skin is thin, smooth, and elastic and is relatively more permeable than that of the adult; consequently, the infant is at greater risk for fluid loss. The subcutaneous layer in the infant is inefficient, not thick, and the sebaceous glands are present but decrease in size and production. Vernix caseosa is not produced after birth. 5. The nurse is bathing an 80-year-old man and notices that his skin is wrinkled, thin, lax, and dry. This finding would be related to which factor in the older adult? a. Increased vascularity of the skin b. Increased numbers of sweat and sebaceous glands c. An increase in elastin and a decrease in subcutaneous fat d. An increased loss of elastin and a decrease in subcutaneous fat D An accumulation of factors place the aging person at risk for skin disease and breakdown: the thinning of the skin, a decrease in vascularity and nutrients, the loss of protective cushioning of the subcutaneous layer, a lifetime of environmental trauma to skin, the social changes of aging, a increasingly sedentary lifestyle, and the chance of immobility. 6. During the aging process, the hair can look gray or white and begin to feel thin and fine. The nurse knows that this occurs because of a decrease in the number of functioning: a. Metrocytes. b. Fungacytes. c. Phagocytes. d. Melanocytes. D 13. A patient comes in for a physical examination and complains of “freezing to death” while waiting for her examination. The nurse notes that her skin is pale and cool and attributes this finding to: a. Venous pooling. b. Peripheral vasodilation. c. Peripheral vasoconstriction. d. Decreased arterial perfusion. C A chilly or air-conditioned environment causes vasoconstriction, which results in false pallor and coolness (see Table 12-1). 14. A patient comes to the clinic and tells the nurse that he has been confined to his recliner chair for approximately 3 days with his feet down and he asks the nurse to evaluate his feet. During the assessment, the nurse might expect to find: a. Pallor b. Coolness c. Distended veins d. Prolonged capillary filling time C Keeping the feet in a dependent position causes venous pooling, resulting in redness, warmth, and distended veins. Prolonged elevation would cause pallor and coolness. Immobilization or prolonged inactivity would cause prolonged capillary filling time (see Table 12-1). 15. A patient is especially worried about an area of skin on her feet that has turned white. The health care provider has told her that her condition is vitiligo. The nurse explains to her that vitiligo is: a. Caused by an excess of melanin pigment b. Caused by an excess of apocrine glands in her feet c. Caused by the complete absence of melanin pigment d. Related to impetigo and can be treated with an ointment C Vitiligo is the complete absence of melanin pigment in patchy areas of white or light skin on the face, neck, hands, feet, body folds, and around orifices—otherwise, the depigmented skin is normal. 16. A patient tells the nurse that he has noticed that one of his moles has started to burn and bleed. When assessing his skin, the nurse pays special attention to the danger signs for pigmented lesions and is concerned with which additional finding? a. Color variation b. Border regularity c. Symmetry of lesions d. Diameter of less than 6 mm A Abnormal characteristics of pigmented lesions are summarized in the mnemonic ABCD: asymmetry of pigmented lesion, border irregularity, color variation, and diameter greater than 6 mm. 17. A patient comes to the clinic and states that he has noticed that his skin is redder than normal. The nurse understands that this condition is due to hyperemia and knows that it can be caused by: a. Decreased amounts of bilirubin in the blood b. Excess blood in the underlying blood vessels c. Decreased perfusion to the surrounding tissues d. Excess blood in the dilated superficial capillaries D Erythema is an intense redness of the skin caused by excess blood (hyperemia) in the dilated superficial capillaries. 18. During a skin assessment, the nurse notices that a Mexican-American patient has skin that is yellowish-brown; however, the skin on the hard and soft palate is pink and the patient’s scleras are not yellow. From this finding, the nurse could probably rule out: a. Pallor b. Jaundice c. Cyanosis d. Iron deficiency B Jaundice is exhibited by a yellow color, which indicates rising levels of bilirubin in the blood. Jaundice is first noticed in the junction of the hard and soft palate in the mouth and in the scleras. 19. A black patient is in the intensive care unit because of impending shock after an accident. The nurse expects to find what characteristics in this patient’s skin? a. Ruddy blue. b. Generalized pallor. c. Ashen, gray, or dull. d. Patchy areas of pallor. C Pallor attributable to shock, with decreased perfusion and vasoconstriction, in black- skinned people will cause the skin to appear ashen, gray, or dull (see Table 12-2). 20. An older adult woman is brought to the emergency department after being found lying on the kitchen floor for 2 days; she is extremely dehydrated. What would the nurse expect to see during the examination? a. Smooth mucous membranes and lips b. Dry mucous membranes and cracked lips c. Pale mucous membranes d. White patches on the mucous membranes B With dehydration, mucous membranes appear dry and the lips look parched and cracked. The other responses are not found in dehydration. 21. A 42-year-old woman complains that she has noticed several small, slightly raised, bright red dots on her chest. On examination, the nurse expects that the spots are probably: a. Anasarca. b. Scleroderma. c. Senile angiomas. d. Latent myeloma. C Cherry (senile) angiomas are small, smooth, slightly raised bright red dots that commonly appear on the trunk of adults over 30 years old. 22. A 65-year-old man with emphysema and bronchitis has come to the clinic for a follow-up appointment. On assessment, the nurse might expect to see which finding? a. Anasarca b. Scleroderma c. Pedal erythema d. Clubbing of the nails D superficial, and thin walled. A wheal is superficial, raised, transient, erythematous, and irregular in shape attributable to edema. A nodule is solid, elevated, hard or soft, and larger than 1 cm. 28. The nurse just noted from the medical record that the patient has a lesion that is confluent in nature. On examination, the nurse expects to find: a. Lesions that run together. b. Annular lesions that have grown together. c. Lesions arranged in a line along a nerve route. d. Lesions that are grouped or clustered together. A Confluent lesions (as with urticaria [hives]) run together. Grouped lesions are clustered together. Annular lesions are circular in nature. Zosteriform lesions are arranged along a nerve route. 29. A patient has had a “terrible itch” for several months that he has been continuously scratching. On examination, the nurse might expect to find: a. A keloid. b. A fissure. c. Keratosis. d. Lichenification. D Lichenification results from prolonged, intense scratching that eventually thickens the skin and produces tightly packed sets of papules. A keloid is a hypertrophic scar. A fissure is a linear crack with abrupt edges, which extends into the dermis; it can be dry or moist. Keratoses are lesions that are raised, thickened areas of pigmentation that appear crusted, scaly, and warty. 30. A physician has diagnosed a patient with purpura. After leaving the room, a nursing student asks the nurse what the physician saw that led to that diagnosis. The nurse should say, “The physician is referring to the: a. “Blue dilation of blood vessels in a star- shaped linear pattern on the legs.” b. “Fiery red, star-shaped marking on the cheek that has a solid circular center.” c. “Confluent and extensive patch of petechiae and ecchymoses on the feet.” d. “Tiny areas of hemorrhage that are less than 2 mm, round, discrete, and dark red in color.” C Purpura is a confluent and extensive patch of petechiae and ecchymoses and a flat macular hemorrhage observed in generalized disorders such as thrombocytopenia and scurvy. The blue dilation of blood vessels in a star-shaped linear pattern on the legs describes a venous lake. The fiery red, star-shaped marking on the cheek that has a solid circular center describes a spider or star angioma. The tiny areas of hemorrhage that are less than 2 mm, round, discrete, and dark red in color describes petechiae. 31. A mother has noticed that her son, who has been to a new babysitter, has some blisters and scabs on his face and buttocks. On examination, the nurse notices moist, thin-roofed vesicles with a thin erythematous base and suspects: a. Eczema. b. Impetigo. c. Herpes zoster. d. Diaper dermatitis. B Impetigo is moist, thin-roofed vesicles with a thin erythematous base and is a contagious bacterial infection of the skin and most common in infants and children. Eczema is characterized by erythematous papules and vesicles with weeping, oozing, and crusts. Herpes zoster (i.e., chickenpox or varicella) is characterized by small, tight vesicles that are shiny with an erythematous base. Diaper dermatitis is characterized by red, moist maculopapular patches with poorly defined borders. 32. The nurse notices that a school-aged child has bluish-white, red-based spots in her mouth that are elevated approximately 1 to 3 mm. What other signs would the nurse expect to find in this patient? a. Pink, papular rash on the face and neck b. Pruritic vesicles over her trunk and neck c. Hyperpigmentation on the chest, abdomen, and back of the arms d. Red-purple, maculopapular, blotchy rash behind the ears and on the face D With measles (rubeola), the examiner assesses a red-purple, blotchy rash on the third or fourth day of illness that appears first behind the ears, spreads over the face, and then over the neck, trunk, arms, and legs. The rash appears coppery and does not blanch. The bluish-white, red- based spots in the mouth are known as Koplik spots. 33. The nurse is assessing the skin of a patient who has acquired immunodeficiency syndrome (AIDS) and notices multiple patchlike lesions on the temple and beard area that are faint pink in color. The nurse recognizes these lesions as: a. Measles (rubeola). b. Kaposi’s sarcoma. c. Angiomas. d. Herpes zoster. B Kaposi’s sarcoma is a vascular tumor that, in the early stages, appears as multiple, patchlike, faint pink lesions over the patient’s temple and beard areas. Measles is characterized by a red-purple maculopapular blotchy rash that appears on the third or fourth day of illness. The rash is first observed behind the ears, spreads over the face, and then spreads over the neck, trunk, arms, and legs. Cherry (senile) angiomas are small (1 to 5 mm), smooth, slightly raised bright red dots that commonly appear on the trunk in all adults over 30 years old. Herpes zoster causes vesicles up to 1 cm in size that are elevated with a cavity containing clear fluid. 34. A 45-year-old farmer comes in for a skin evaluation and complains of hair loss on his head. His hair seems to be breaking off in patches, and he notices some scaling on his head. The nurse begins the examination suspecting: a. Tinea capitis. b. Folliculitis. c. Toxic alopecia. d. Seborrheic dermatitis. A Tinea capitis is rounded patchy hair loss on the scalp, leaving broken-off hairs, pustules, and scales on the skin, and is caused by a fungal infection. Lesions are fluorescent under a Wood light and are usually observed in children and farmers; tinea capitis is highly contagious. (See Table 12-12, Abnormal Conditions of Hair, for descriptions of the other terms.) 35. A mother brings her child into the clinic for an examination of the scalp and hair. She states that the child has developed irregularly shaped patches with broken-off, stublike hair in some places; she is worried that this condition could be some form of premature baldness. The nurse tells her that it is: a. Folliculitis that can be treated with an antibiotic. b. Traumatic alopecia that can be treated with antifungal medications. c. Tinea capitis that is highly contagious and needs immediate attention. that are common in the outer sclera of dark-skinned persons. 41. The nurse is assessing for inflammation in a dark-skinned person. Which technique is the best? a. Assessing the skin for cyanosis and swelling b. Assessing the oral mucosa for generalized erythema c. Palpating the skin for edema and increased warmth d. Palpating for tenderness and local areas of ecchymosis C Because inflammation cannot be seen in dark-skinned persons, palpating the skin for increased warmth, for taut or tightly pulled surfaces that may be indicative of edema, and for a hardening of deep tissues or blood vessels is often necessary. 42. A few days after a summer hiking trip, a 25-year-old man comes to the clinic with a rash. On examination, the nurse notes that the rash is red, macular, with a bull’s eye pattern across his midriff and behind his knees. The nurse suspects: a. Rubeola. b. Lyme disease. c. Allergy to mosquito bites. d. Rocky Mountain spotted fever. B Lyme disease occurs in people who spend time outdoors in May through September. The first disease state exhibits the distinctive bull’s eye and a red macular or papular rash that radiates from the site of the tick bite with some central clearing. The rash spreads 5 cm or larger, and is usually in the axilla, midriff, inguinal, or behind the knee, with regional lymphadenopathy. 43. A 52-year-old woman has a papule on her nose that has rounded, pearly borders and a central red ulcer. She said she first noticed it several months ago and that it has slowly grown larger. The nurse suspects which condition? a. Acne b. Basal cell carcinoma c. Melanoma d. Squamous cell carcinoma B Basal cell carcinoma usually starts as a skin-colored papule that develops rounded, pearly borders with a central red ulcer. It is the most common form of skin cancer and grows slowly. This description does not fit acne lesions. (See Table 12-11 for descriptions of melanoma and squamous cell carcinoma.) 44. A father brings in his 2-month-old infant to the clinic because the infant has had diarrhea for the last 24 hours. He says his baby has not been able to keep any formula down and that the diarrhea has been at least every 2 hours. The nurse suspects dehydration. The nurse should test skin mobility and turgor over the infant’s: a. Sternum. b. Forehead. c. Forearms. d. Abdomen. D Mobility and turgor are tested over the abdomen in an infant. Poor turgor, or tenting, indicates dehydration or malnutrition. The other sites are not appropriate for checking skin turgor in an infant. 45. A semiconscious woman is brought to the emergency department after she was found on the floor in her kitchen. Her face, nail beds, lips, and oral mucosa are a bright cherry-red color. The nurse suspects that this coloring is due to: a. Polycythemia. b. Carbon monoxide poisoning. c. Carotenemia. d. Uremia. B A bright cherry-red coloring in the face, upper torso, nail beds, lips, and oral mucosa appears in cases of carbon monoxide poisoning. 46. A patient has been admitted for severe psoriasis. The nurse expects to see what finding in the patient’s fingernails? a. Splinter hemorrhages b. Paronychia c. Pitting d. Beau lines C Sharply defined pitting and crumbling of the nails, each with distal detachment characterize pitting nails and are associated with psoriasis. (See Table 12-13 for descriptions of the other terms.) MULTIPLE RESPONSE 1. The nurse is preparing for a certification course in skin care and needs to be familiar with the various lesions that may be identified on assessment of the skin. Which of the following definitions are correct? Select all that apply. a. Petechiae: Tiny punctate hemorrhages, 1 to 3 mm, round and discrete, dark red, purple, or brown in color b. Bulla: Elevated, circumscribed lesion filled with turbid fluid (pus) c. Papule: Hypertrophic scar d. Vesicle: Known as a friction blister e. Nodule: Solid, elevated, and hard or soft growth that is larger than 1 cm A, D, E A pustule is an elevated, circumscribed lesion filled with turbid fluid (pus). A hypertrophic scar is a keloid. A bulla is larger than 1 cm and contains clear fluid. A papule is solid and elevated but measures less than 1 cm. 2. A patient has been admitted to a hospital after the staff in the nursing home noticed a pressure ulcer in his sacral area. The nurse examines the pressure ulcer and determines that it is a stage II ulcer. Which of these findings are characteristic of a stage II pressure ulcer? Select all that apply. a. Intact skin appears red but is not broken. b. Partial thickness skin erosion is observed with a loss of epidermis or dermis. c. Ulcer extends into the subcutaneous tissue. d. Localized redness in light skin will blanch with fingertip pressure. e. Open blister areas have a red-pink wound bed. f. Patches of eschar cover parts of the wound. B, E Stage I pressure ulcers have intact skin that appears red but is not broken, and localized c. XII; percussing the sternomastoid and submandibular neck muscles d. XII; assessing for a positive Romberg sign B The major neck muscles are the sternomastoid and the trapezius. They are innervated by CN XI, the spinal accessory. The innervated muscles assist with head rotation and head flexion, movement of the shoulders, and extension and turning of the head. 7. When examining a patient’s CN function, the nurse remembers that the muscles in the neck that are innervated by CN XI are the: a. Sternomastoid and trapezius. b. Spinal accessory and omohyoid. c. Trapezius and sternomandibular. d. Sternomandibular and spinal accessory. A The major neck muscles are the sternomastoid and the trapezius. They are innervated by CN XI, the spinal accessory. 8. A patient’s laboratory data reveal an elevated thyroxine (T4) level. The nurse would proceed with an examination of the gland. a. Thyroid b. Parotid c. Adrenal d. Parathyroid A The thyroid gland is a highly vascular endocrine gland that secretes T4 and triiodothyronine (T3). The other glands do not secrete T4. 9. A patient says that she has recently noticed a lump in the front of her neck below her “Adam’s apple” that seems to be getting bigger. During the assessment, the finding that leads the nurse to suspect that this may not be a cancerous thyroid nodule is that the lump (nodule): a. Is tender. b. Is mobile and not hard. c. Disappears when the patient smiles. d. Is hard and fixed to the surrounding structures. B Painless, rapidly growing nodules may be cancerous, especially the appearance of a single nodule in a young person. However, cancerous nodules tend to be hard and fixed to surrounding structures, not mobile. 10. The nurse notices that a patient’s submental lymph nodes are enlarged. In an effort to identify the cause of the node enlargement, the nurse would assess the patient’s: a. Infraclavicular area. b. Supraclavicular area. c. Area distal to the enlarged node. d. Area proximal to the enlarged node. D When nodes are abnormal, the nurse should check the area into which they drain for the source of the problem. The area proximal (upstream) to the location of the abnormal node should be explored. 11. The nurse is aware that the four areas in the body where lymph nodes are accessible are the: a. Head, breasts, groin, and abdomen. b. Arms, breasts, inguinal area, and legs. c. Head and neck, arms, breasts, and axillae. d. Head and neck, arms, inguinal area, and axillae. D Nodes are located throughout the body, but they are accessible to examination only in four areas: head and neck, arms, inguinal region, and axillae. 12. A mother brings her newborn in for an assessment and asks, “Is there something wrong with my baby? His head seems so big.” Which statement is true regarding the relative proportions of the head and trunk of the newborn? a. At birth, the head is one fifth the total length. b. Head circumference should be greater than chest circumference at birth. c. The head size reaches 90% of its final size when the child is 3 years old. d. When the anterior fontanel closes at 2 months, the head will be more proportioned to the body. B The nurse recognizes that during the fetal period, head growth predominates. Head size is greater than chest circumference at birth, and the head size grows during childhood, reaching 90% of its final size when the child is age 6 years. 13. A patient, an 85-year-old woman, is complaining about the fact that the bones in her face have become more noticeable. What explanation should the nurse give her? a. Diets low in protein and high in carbohydrates may cause enhanced facial bones. b. Bones can become more noticeable if the person does not use a dermatologically approved moisturizer. c. More noticeable facial bones are probably due to a combination of factors related to aging, such as decreased elasticity, subcutaneous fat, and moisture in her skin. d. Facial skin becomes more elastic with age. This increased elasticity causes the skin to be more taught, drawing attention to the facial bones. C The facial bones and orbits appear more prominent in the aging adult, and the facial skin sags, which is attributable to decreased elasticity, decreased subcutaneous fat, and decreased moisture in the skin. 14. A patient reports excruciating headache pain on one side of his head, especially around his eye, forehead, and cheek that has lasted approximately to 2 hours, occurring once or twice each day. The nurse should suspect: a. Hypertension. b. Cluster headaches. c. Tension headaches. d. Migraine headaches. B Cluster headaches produce pain around the eye, temple, forehead, and cheek and are unilateral and always on the same side of the head. They are excruciating and occur once or twice per day and last to 2 hours each. 15. A patient complains that while studying for an examination he began to notice a severe headache in the frontotemporal area of his head that is throbbing and is somewhat relieved when he lies down. He tells the nurse that his mother also had these headaches. The nurse suspects that he may be suffering from: d. Atrophied nodular thyroid gland A T4 and T3 are thyroid hormones that stimulate the rate of cellular metabolism, resulting in tachycardia. With an enlarged thyroid gland as in hyperthyroidism, the nurse might expect to find diffuse enlargement (goiter) or a nodular lump but not an atrophied gland. Dyspnea and constipation are not findings associated with hyperthyroidism. 22. A visitor from Poland who does not speak English seems to be somewhat apprehensive about the nurse examining his neck. He would probably be more comfortable with the nurse examining his thyroid gland from: a. Behind with the nurse’s hands placed firmly around his neck. b. The side with the nurse’s eyes averted toward the ceiling and thumbs on his neck. c. The front with the nurse’s thumbs placed on either side of his trachea and his head tilted forward. d. The front with the nurse’s thumbs placed on either side of his trachea and his head tilted backward. C Examining this patient’s thyroid gland from the back may be unsettling for him. It would be best to examine his thyroid gland using the anterior approach, asking him to tip his head forward and to the right and then to the left. 23. A patient’s thyroid gland is enlarged, and the nurse is preparing to auscultate the thyroid gland for the presence of a bruit. A bruit is a sound that is heard best with the of the stethoscope. a. Low gurgling; diaphragm b. Loud, whooshing, blowing; bell c. Soft, whooshing, pulsatile; bell d. High-pitched tinkling; diaphragm C If the thyroid gland is enlarged, then the nurse should auscultate it for the presence of a bruit, which is a soft, pulsatile, whooshing, blowing sound heard best with the bell of the stethoscope. 24. The nurse notices that an infant has a large, soft lump on the side of his head and that his mother is very concerned. She tells the nurse that she noticed the lump approximately 8 hours after her baby’s birth and that it seems to be getting bigger. One possible explanation for this is: a. Hydrocephalus. b. Craniosynostosis. c. Cephalhematoma. d. Caput succedaneum. C A cephalhematoma is a subperiosteal hemorrhage that is the result of birth trauma. It is soft, fluctuant, and well defined over one cranial bone. It appears several hours after birth and gradually increases in size. 25. A mother brings in her newborn infant for an assessment and tells the nurse that she has noticed that whenever her newborn’s head is turned to the right side, she straightens out the arm and leg on the same side and flexes the opposite arm and leg. After observing this on examination, the nurse tells her that this reflex is: a. Abnormal and is called the atonic neck reflex. b. Normal and should disappear by the first year of life. c. Normal and is called the tonic neck reflex, which should disappear between 3 and 4 months of age. d. Abnormal. The baby should be flexing the arm and leg on the right side of his body when the head is turned to the right. C By 2 weeks, the infant shows the tonic neck reflex when supine and the head is turned to one side (extension of same arm and leg, flexion of opposite arm and leg). The tonic neck reflex disappears between 3 and 4 months of age. 26. During an admission assessment, the nurse notices that a male patient has an enlarged and rather thick skull. The nurse suspects acromegaly and would further assess for: a. Exophthalmos. b. Bowed long bones. c. Coarse facial features. d. Acorn-shaped cranium. C Acromegaly is excessive secretion of growth hormone that creates an enlarged skull and thickened cranial bones. Patients will have elongated heads, massive faces, prominent noses and lower jaws, heavy eyebrow ridges, and coarse facial features. Exophthalmos is associated with hyperthyroidism. Bowed long bones and an acorn-shaped cranium result from Paget disease. 27. When examining children affected with Down syndrome (trisomy 21), the nurse looks for the possible presence of: a. Ear dysplasia. b. Long, thin neck. c. Protruding thin tongue. d. Narrow and raised nasal bridge. A With the chromosomal aberration trisomy 21, also known as Down syndrome, head and face characteristics may include upslanting eyes with inner epicanthal folds, a flat nasal bridge, a small broad flat nose, a protruding thick tongue, ear dysplasia, a short broad neck with webbing, and small hands with a single palmar crease. 28. A patient visits the clinic because he has recently noticed that the left side of his mouth is paralyzed. He states that he cannot raise his eyebrow or whistle. The nurse suspects that he has: a. Cushing syndrome. b. Parkinson disease. c. Bell palsy. d. Experienced a cerebrovascular accident (CVA) or stroke. D With an upper motor neuron lesion, as with a CVA, the patient will have paralysis of lower facial muscles, but the upper half of the face will not be affected owing to the intact nerve from the unaffected hemisphere. The person is still able to wrinkle the forehead and close the eyes. (See Table 13-4, Abnormal Facial Appearances with Chronic Illnesses, for descriptions of the other responses.) 29. A woman comes to the clinic and states, “I’ve been sick for so long! My eyes have gotten so puffy, and my eyebrows and hair have become coarse and dry.” The nurse will assess for other signs and symptoms of: a. Cachexia. b. Parkinson syndrome. c. Myxedema. d. Scleroderma. C Myxedema (hypothyroidism) is a deficiency of thyroid hormone that, when severe, causes a nonpitting edema or myxedema. The patient will have a puffy edematous face, especially around the eyes (periorbital edema); coarse facial features; dry skin; and dry, c. Is exhibiting a normal enlargement of the thyroid gland during pregnancy. d. Needs further testing for possible thyroid cancer. C The thyroid gland enlarges slightly during pregnancy because of hyperplasia of the tissue and increased vascularity. 37. During an examination, the nurse knows that the best way to palpate the lymph nodes in the neck is described by which statement? a. Using gentle pressure, palpate with both hands to compare the two sides. b. Using strong pressure, palpate with both hands to compare the two sides. c. Gently pinch each node between one’s thumb and forefinger, and then move down the neck muscle. d. Using the index and middle fingers, gently palpate by applying pressure in a rotating pattern. A Using gentle pressure is recommended because strong pressure can push the nodes into the neck muscles. Palpating with both hands to compare the two sides symmetrically is usually most efficient. 38. During a well-baby checkup, a mother is concerned because her 2-month-old infant cannot hold her head up when she is pulled to a sitting position. Which response by the nurse is appropriate? a. “Head control is usually achieved by 4 months of age.” b. “You shouldn’t be trying to pull your baby up like that until she is older.” c. “Head control should be achieved by this time.” d. “This inability indicates possible nerve damage to the neck muscles.” A Head control is achieved by 4 months when the baby can hold the head erect and steady when pulled to a vertical position. The other responses are not appropriate. 39. During an examination of a 3-year-old child, the nurse notices a bruit over the left temporal area. The nurse should: a. Continue the examination because a bruit is a normal finding for this age. b. Check for the bruit again in 1 hour. c. Notify the parents that a bruit has been detected in their child. d. Stop the examination, and notify the physician. A Bruits are common in the skull in children under 4 or 5 years of age and in children with anemia. They are systolic or continuous and are heard over the temporal area. 40. During an examination, the nurse finds that a patient’s left temporal artery is tortuous and feels hardened and tender, compared with the right temporal artery. The nurse suspects which condition? a. Crepitation b. Mastoiditis c. Temporal arteritis d. Bell palsy C With temporal arteritis, the artery appears more tortuous and feels hardened and tender. These assessment findings are not consistent with the other responses. MULTIPLE RESPONSE 1. The nurse is assessing a 1-month-old infant at his well-baby checkup. Which assessment findings are appropriate for this age? Select all that apply. a. Head circumference equal to chest circumference b. Head circumference greater than chest circumference c. Head circumference less than chest circumference d. Fontanels firm and slightly concave e. Absent tonic neck reflex f. Nonpalpable cervical lymph nodes B, D, F An infant’s head circumference is larger than the chest circumference. At age 2 years, both measurements are the same. During childhood, the chest circumference grows to exceed the head circumference by 5 to 7 cm. The fontanels should feel firm and slightly concave in the infant, and they should close by age 9 months. The tonic neck reflex is present until between 3 and 4 months of age, and cervical lymph nodes are normally nonpalpable in an infant. Chapter 14 1. When examining the eye, the nurse notices that the patient’s eyelid margins approximate completely. The nurse recognizes that this assessment finding: a. Is expected. b. May indicate a problem with extraocular muscles. c. May result in problems with tearing. d. Indicates increased intraocular pressure. A The palpebral fissure is the elliptical open space between the eyelids, and, when closed, the lid margins approximate completely, which is a normal finding. 2. During ocular examinations, the nurse keeps in mind that movement of the extraocular muscles is: a. Decreased in the older adult. b. Impaired in a patient with cataracts. c. Stimulated by cranial nerves (CNs) I and II. d. Stimulated by CNs III, IV, and VI. D Movement of the extraocular muscles is stimulated by three CNs: III, IV, and VI. 3. The nurse is performing an external eye examination. Which statement regarding the outer layer of the eye is true? a. The outer layer of the eye is very sensitive to touch. b. The outer layer of the eye is darkly pigmented to prevent light from reflecting internally. c. The trigeminal nerve (CN V) and the trochlear nerve (CN IV) are stimulated response to bright light. D The pupillary light reflex is the normal constriction of the pupils when bright light shines on the retina. The other responses are not correct. 9. A mother asks when her newborn infant’s eyesight will be developed. The nurse should reply: a. “Vision is not totally developed until 2 years of age.” b. “Infants develop the ability to focus on an object at approximately 8 months of age.” c. “By approximately 3 months of age, infants develop more coordinated eye movements and can fixate on an object.” d. “Most infants have uncoordinated eye movements for the first year of life.” C Eye movements may be poorly coordinated at birth, but by 3 to 4 months of age, the infant should establish binocularity and should be able to fixate simultaneously on a single image with both eyes. 10. The nurse is reviewing in age-related changes in the eye for a class. Which of these physiologic changes is responsible for presbyopia? a. Degeneration of the cornea b. Loss of lens elasticity c. Decreased adaptation to darkness d. Decreased distance vision abilities B The lens loses elasticity and decreases its ability to change shape to accommodate for near vision. This condition is called presbyopia. 11. Which of these assessment findings would the nurse expect to see when examining the eyes of a black patient? a. Increased night vision b. Dark retinal background c. Increased photosensitivity d. Narrowed palpebral fissures B An ethnically based variability in the color of the iris and in retinal pigmentation exists, with darker irides having darker retinas behind them. 12. A 52-year-old patient describes the presence of occasional floaters or spots moving in front of his eyes. The nurse should: a. Examine the retina to determine the number of floaters. b. Presume the patient has glaucoma and refer him for further testing. c. Consider these to be abnormal findings, and refer him to an ophthalmologist. d. Know that floaters are usually insignificant and are caused by condensed vitreous fibers. D Floaters are a common sensation with myopia or after middle age and are attributable to condensed vitreous fibers. Floaters or spots are not usually significant, but the acute onset of floaters may occur with retinal detachment. 13. The nurse is preparing to assess the visual acuity of a 16-year-old patient. How should the nurse proceed? a. Perform the confrontation test. b. Ask the patient to read the print on a handheld Jaeger card. c. Use the Snellen chart positioned 20 feet away from the patient. d. Determine the patient’s ability to read newsprint at a distance of 12 to 14 inches. C The Snellen alphabet chart is the most commonly used and most accurate measure of visual acuity. The confrontation test is a gross measure of peripheral vision. The Jaeger card or newspaper tests are used to test near vision. 14. A patient’s vision is recorded as 20/30 when the Snellen eye chart is used. The nurse interprets these results to indicate that: a. At 30 feet the patient can read the entire chart. b. The patient can read at 20 feet what a person with normal vision can read at 30 feet. c. The patient can read the chart from 20 feet in the left eye and 30 feet in the right eye. d. The patient can read from 30 feet what a person with normal vision can read from 20 feet. B The top number indicates the distance the person is standing from the chart; the denominator gives the distance at which a normal eye can see. 15. A patient is unable to read even the largest letters on the Snellen chart. The nurse should take which action next? a. Refer the patient to an ophthalmologist or optometrist for further evaluation. b. Assess whether the patient can count the nurse’s fingers when they are placed in front of his or her eyes. c. Ask the patient to put on his or her reading glasses and attempt to read the Snellen chart again. d. Shorten the distance between the patient and the chart until the letters are seen, and record that distance. D If the person is unable to see even the largest letters when standing 20 feet from the chart, then the nurse should shorten the distance to the chart until the letters are seen, and record that distance (e.g., “10/200”). If visual acuity is even lower, then the nurse should assess whether the person can count fingers when they are spread in front of the eyes or can distinguish light perception from a penlight. If vision is poorer than 20/30, then a referral to an ophthalmologist or optometrist is necessary, but the nurse must first assess the visual acuity. 16. A patient’s vision is recorded as 20/80 in each eye. The nurse interprets this finding to mean that the patient: a. Has poor vision. b. Has acute vision. c. Has normal vision. d. Is presbyopic. A Normal visual acuity is 20/20 in each eye; the larger the denominator, the poorer the vision. 17. When performing the corneal light reflex assessment, the nurse notes that the light is reflected at 2 o’clock in each eye. The nurse should: 23. The nurse is assessing a patient’s eyes for the accommodation response and would expect to see which normal finding? a. Dilation of the pupils b. Consensual light reflex c. Conjugate movement of the eyes d. Convergence of the axes of the eyes D The accommodation reaction includes pupillary constriction and convergence of the axes of the eyes. The other responses are not correct. 24. In using the ophthalmoscope to assess a patient’s eyes, the nurse notices a red glow in the patient’s pupils. On the basis of this finding, the nurse would: a. Suspect that an opacity is present in the lens or cornea. b. Check the light source of the ophthalmoscope to verify that it is functioning. c. Consider the red glow a normal reflection of the ophthalmoscope light off the inner retina. d. Continue with the ophthalmoscopic examination, and refer the patient for further evaluation. C The red glow filling the person’s pupil is the red reflex and is a normal finding caused by the reflection of the ophthalmoscope light off the inner retina. The other responses are not correct. 25. The nurse is examining a patient’s retina with an ophthalmoscope. Which finding is considered normal? a. Optic disc that is a yellow-orange color b. Optic disc margins that are blurred around the edges c. Presence of pigmented crescents in the macular area d. Presence of the macula located on the nasal side of the retina A The optic disc is located on the nasal side of the retina. Its color is a creamy yellow- orange to a pink, and the edges are distinct and sharply demarcated, not blurred. A pigmented crescent is black and is due to the accumulation of pigment in the choroid. 26. A 2-week-old infant can fixate on an object but cannot follow a light or bright toy. The nurse would: a. Consider this a normal finding. b. Assess the pupillary light reflex for possible blindness. c. Continue with the examination, and assess visual fields. d. Expect that a 2-week-old infant should be able to fixate and follow an object. A By 2 to 4 weeks an infant can fixate on an object. By the age of 1 month, the infant should fixate and follow a bright light or toy. 27. The nurse is assessing color vision of a male child. Which statement is correct? The nurse should: a. Check color vision annually until the age of 18 years. b. Ask the child to identify the color of his or her clothing. c. Test for color vision once between the ages of 4 and 8 years. d. Begin color vision screening at the child’s 2- year checkup. C Test boys only once for color vision between the ages of 4 and 8 years. Color vision is not tested in girls because it is rare in girls. Testing is performed with the Ishihara test, which is a series of polychromatic cards. 28. The nurse is performing an eye-screening clinic at a daycare center. When examining a 2- year- old child, the nurse suspects that the child has a “lazy eye” and should: a. Examine the external structures of the eye. b. Assess visual acuity with the Snellen eye chart. c. Assess the child’s visual fields with the confrontation test. d. Test for strabismus by performing the corneal light reflex test. D Testing for strabismus is done by performing the corneal light reflex test and the cover test. The Snellen eye chart and confrontation test are not used to test for strabismus. 29. The nurse is performing an eye assessment on an 80-year-old patient. Which of these findings is considered abnormal? a. Decrease in tear production b. Unequal pupillary constriction in response to light c. Presence of arcus senilis observed around the cornea d. Loss of the outer hair on the eyebrows attributable to a decrease in hair follicles B Pupils are small in the older adult, and the pupillary light reflex may be slowed, but pupillary constriction should be symmetric. The assessment findings in the other responses are considered normal in older persons. 30. The nurse notices the presence of periorbital edema when performing an eye assessment on a 70-year-old patient. The nurse should: a. Check for the presence of exophthalmos. b. Suspect that the patient has hyperthyroidism. c. Ask the patient if he or she has a history of heart failure. d. Assess for blepharitis, which is often associated with periorbital edema. C Periorbital edema occurs with local infections, crying, and systemic conditions such as heart failure, renal failure, allergy, and hypothyroidism. Periorbital edema is not associated with blepharitis. 31. When a light is directed across the iris of a patient’s eye from the temporal side, the nurse is assessing for: a. Drainage from dacryocystitis. b. Presence of conjunctivitis over the iris. c. Presence of shadows, which may indicate glaucoma. d. Scattered light reflex, which may be indicative of cataracts.
Docsity logo



Copyright © 2024 Ladybird Srl - Via Leonardo da Vinci 16, 10126, Torino, Italy - VAT 10816460017 - All rights reserved